Proving the Lorentz invariance of an integration measure? QFT related?

This means that d^4k is a Lorentz invariant measure of four-momentum. As for the theta function, it ensures that the integral only includes positive values of k0, which is necessary for the expression to be Lorentz invariant.
  • #1
jeebs
325
4
qftproblem21.jpg


So, first off, I'm thinking Lorentz invariant quantities are the same in any inertial frames S and S' regardless of their relative velocity.

I'm thinking I need to show that
[itex]\frac{d^3k}{(2\pi)^32E(\vec{k})} = \frac{d^3k'}{(2\pi)^32E'(\vec{k'})}[/itex] where the primed & unprimed quantities denote different frames.

We also have [itex] E(\vec{k}) = \sqrt{\vec{k}^2 + m^2}[/itex] in the denominator. This isn't Lorentz invariant - the mass term is, but the 3 - momentum [itex]\vec{p} = \hbar \vec{k}[/itex] is not, therefore E is not.

[itex]E \neq E'[/itex] where [itex] E'(\vec{k'}) = \sqrt{\vec{k}'^2 + m^2}[/itex].

This is making me think that neither E nor d3k are Lorentz invariant, so that when they are used in this fraction, the Lorentz invariance from each one somehow cancels out overall.

Then we come to the hint. I 'm really not sure what to make of this, I mean, I can write [itex]\delta(x^2 - x_0^2) = \frac{1}{2|x|}(\delta(x-x_0) + \delta(x+x_0))[/itex] with k's and m's: [itex]\delta(k^2 - m^2) = \frac{1}{2|k|}(\delta(k-m) + \delta(k+m))[/itex]
to get an apparently Lorentz invariant expression: [itex] \frac{d^4k}{(2\pi)^3}\delta(k^2 - m^2)\theta(k_0) = \frac{d^4k}{(2\pi)^3}\frac{1}{2|k|}(\delta(k-m) + \delta(k+m))\theta(k_0) [/itex]
but I'm not sure where this gets me, other than guessing that
[itex] \delta(k^2 - m^2)\theta(k_0) [/itex] is also Lorentz invariant seeing as I am told d4k is.

I have no idea how to proceed with this. I don't know what to make of the delta functions either.
 
Physics news on Phys.org
  • #2
You should recognize that [itex]d^4k\ \delta(k^2-m^2)\theta(k_0)[/itex] is manifestly Lorentz invariant because [itex]k^2 = k^\mu k_\mu[/itex] is invariant.

The idea here is to note that [itex]k^2-m^2 = k_0^2-\mathbf{k}^2-m^2[/itex] and to do the integral over k0.

Intuitively, you can think of the volume [itex]d^3k[/itex] being reduced by a factor [itex]\gamma[/itex] due to length contraction while E(k) is reduced by the same factor due to time dilation, so their ratio is invariant.
 
  • #3
vela said:
You should recognize that [itex]d^4k\ \delta(k^2-m^2)\theta(k_0)[/itex] is manifestly Lorentz invariant because [itex]k^2 = k^\mu k_\mu[/itex] is invariant.

right, so, if I'm not mistaken, [itex]k^\mu = (k_0, k_1, k_2, k_3)[/itex] and [itex]k_\mu = (k_0, -k_1, -k_2, -k_3)[/itex]. So, [itex]k^2 = k^\mu k_\mu = (k_0, k_1, k_2, k_3).(k_0, -k_1, -k_2, -k_3) = k_0^2 - k_1^2 -k_2^2 -k_3^2 = k_0^2 - \vec{k}^2[/itex].
Is there some reason why I should instantly recognise this as an invariant quantity?
Do you always get an invariant quantity if you take any 4-vector and do this with it?
Also, what am I supposed to make of the [itex]\theta(k_0)[/itex] or the d4k? What about them says Lorentz invariant?

vela said:
The idea here is to note that [itex]k^2-m^2 = k_0^2-\mathbf{k}^2-m^2[/itex] and to do the integral over k0.

I'm meant to be computing [itex]\frac{1}{(2\pi)^3}∫ \delta(k_0^2 - \vec{k}^2 - m^2)\theta(k_0)d^4k[/itex] right?
I'm struggling to be change the [itex]d^4k[/itex] into something involving a dk0.
Can I start off by saying [itex]dk^\mu = (dk_0, dk_1, dk_2, dk_3)[/itex]?

Actually, can I say that [itex] d^4k = d^3k dk_0 [/itex]?
 
Last edited:
  • #4
jeebs said:
right, so, if I'm not mistaken, [itex]k^\mu = (k_0, k_1, k_2, k_3)[/itex] and [itex]k_\mu = (k_0, -k_1, -k_2, -k_3)[/itex]. So, [itex]k^2 = k^\mu k_\mu = (k_0, k_1, k_2, k_3).(k_0, -k_1, -k_2, -k_3) = k_0^2 - k_1^2 -k_2^2 -k_3^2 = k_0^2 - \vec{k}^2[/itex].
Is there some reason why I should instantly recognise this as an invariant quantity?
Do you always get an invariant quantity if you take any 4-vector and do this with it?
Yes, the product of any two four-vectors, including a four-vector with itself, is an invariant. It's analogous to the regular dot product being invariant under rotations.
Also, what am I supposed to make of the [itex]\theta(k_0)[/itex] or the d4k? What about them says Lorentz invariant?
Think about the coordinate transformation [itex]k'^\mu = \Lambda^\mu{}_\nu k^\nu[/itex] where [itex]\Lambda^\mu{}_\nu[/itex] is a Lorentz transformation. How are the volume elements related? What about the signs of k0 of k'0?
I'm meant to be computing [itex]\frac{1}{(2\pi)^3}∫ \delta(k_0^2 - \vec{k}^2 - m^2)\theta(k_0)d^4k[/itex] right?
I'm struggling to be change the [itex]d^4k[/itex] into something involving a dk0.
Can I start off by saying [itex]dk^\mu = (dk_0, dk_1, dk_2, dk_3)[/itex]?

Actually, can I say that [itex] d^4k = d^3k dk_0 [/itex]?
Yes, [itex] d^4k = d^3k\:dk_0 [/itex]
 
  • #5
Are they supposed to be Lorentz invariant?

I would approach this problem by first understanding the concept of Lorentz invariance and its relevance in quantum field theory (QFT). Lorentz invariance refers to the principle that the laws of physics should be the same in all inertial frames of reference. In other words, the physical quantities and measurements should not depend on the observer's velocity or frame of reference.

In QFT, it is important to ensure that the integration measures used to calculate physical quantities are also Lorentz invariant. This is because the integration measure is used to integrate over all possible momentum states of particles, and if it is not Lorentz invariant, it can lead to incorrect results.

In this case, we are trying to prove the Lorentz invariance of the integration measure \frac{d^3k}{(2\pi)^32E(\vec{k})}. As you correctly pointed out, the energy term E(\vec{k}) is not Lorentz invariant due to the momentum term \vec{p} = \hbar \vec{k}. However, we can use the hint provided to us and write the measure in terms of delta functions.

The delta function \delta(k^2 - m^2) can be interpreted as a measure of the energy of a particle with momentum \vec{k} and mass m. This delta function is Lorentz invariant, as it only depends on the magnitude of the momentum \vec{k} and not its direction. Therefore, when we multiply this delta function with the Lorentz non-invariant measure \frac{d^3k}{(2\pi)^32E(\vec{k})}, the non-invariance of the energy term cancels out, and we are left with a Lorentz invariant measure.

Furthermore, the theta function \theta(k_0) ensures that we only integrate over positive energy states, which is also Lorentz invariant. Therefore, we can conclude that the expression \frac{d^4k}{(2\pi)^3}\delta(k^2 - m^2)\theta(k_0) is Lorentz invariant.

In summary, the use of delta functions in the integration measure ensures that it is Lorentz invariant, despite the non-invariance of the energy term. This is a crucial step in proving the Lorentz invariance of the integration measure in QFT
 

Related to Proving the Lorentz invariance of an integration measure? QFT related?

1. What is Lorentz invariance?

Lorentz invariance is a fundamental principle in physics that states that the laws of physics should be the same for all inertial observers, regardless of their relative motion.

2. Why is proving Lorentz invariance important in quantum field theory (QFT)?

In QFT, Lorentz invariance is a crucial aspect of the theory as it ensures that the theory is consistent with special relativity and the principles of causality and locality.

3. How is Lorentz invariance related to the integration measure in QFT?

In QFT, the integration measure is used to integrate over all possible field configurations. Proving Lorentz invariance of this measure ensures that the integration is performed in a consistent manner for all inertial observers.

4. What is the process for proving the Lorentz invariance of an integration measure?

The process involves showing that the integration measure is invariant under Lorentz transformations, which can be done using mathematical techniques such as Lorentz covariance and Lorentz invariance of the action functional.

5. Are there any experimental tests that have confirmed the Lorentz invariance of an integration measure?

While there have been numerous experimental tests of Lorentz invariance in other areas of physics, there are currently no direct experimental tests of the Lorentz invariance of an integration measure in QFT. However, the consistency of QFT with special relativity and the success of its predictions in experimental tests provide strong indirect evidence for the Lorentz invariance of the integration measure.

Similar threads

  • Advanced Physics Homework Help
Replies
4
Views
374
  • Advanced Physics Homework Help
Replies
15
Views
2K
  • Advanced Physics Homework Help
Replies
11
Views
2K
  • Advanced Physics Homework Help
Replies
7
Views
1K
  • Advanced Physics Homework Help
Replies
1
Views
601
  • Advanced Physics Homework Help
Replies
2
Views
1K
Replies
27
Views
2K
  • Advanced Physics Homework Help
Replies
7
Views
1K
  • Advanced Physics Homework Help
Replies
1
Views
931
  • Advanced Physics Homework Help
Replies
6
Views
369
Back
Top